Non-commutativity of unit polar bases

  • I
  • Thread starter chartery
  • Start date
  • Tags
    Chain rule
  • #1
chartery
31
4
I'm having trouble(s) showing that unit polar bases do not commute.

Adapting <https://math.stackexchange.com/questions/3288981>

taking: ##\hat{\theta} = \frac{1}{r}\frac{\partial }{\partial \theta} ( =\frac{1}{r}\overrightarrow{e}_{\theta})##
then ##\hat{r}\hat{\theta} = \frac{\partial }{\partial r}\left( \frac{1}{r} \frac{\partial f}{\partial \theta}\right)## by chain rule will differ from ##\hat{\theta}\hat{r} = \frac{1}{r}\frac{\partial }{\partial \theta}\left( \frac{\partial f}{\partial r}\right)##
so far so good, but...in <https://physics.stackexchange.com/questions/198280> (first answer)
(via ##\frac{\partial ^{2}x}{\partial \hat{r}\partial \hat{\theta}} vs \frac{\partial ^{2}x}{\partial \hat{\theta}\partial \hat{r}}##)

##\overrightarrow{e_{\hat{r}}}=\overrightarrow{e_{r}}## seems to justify ##(\frac{\partial }{\partial \hat{r}}\left( -sin\theta \right)= )##, ##\frac{\partial }{\partial \hat{r}}\left( -\frac{y}{r} \right)=\frac{\partial }{\partial r}\left( -\frac{y}{r} \right)##
but I don't see how to apply ##\frac{\partial }{\partial \hat{\theta}}## in ##\frac{\partial }{\partial \hat{\theta}}\left( cos\theta \right)##Finally, when just multiplying out the commutator of Cartesian versions, everything seems to cancel:
##[\hat{r},\hat{\theta}] = \left( cos\theta\overrightarrow{x}+sin\theta\overrightarrow{y} \right)\left( -sin\theta\overrightarrow{x}+cos\theta\overrightarrow{y} \right)-\left( -sin\theta\overrightarrow{x}+cos\theta\overrightarrow{y} \right)\left( cos\theta\overrightarrow{x}+sin\theta\overrightarrow{y} \right)## =0??Evidently, my grasp of the underlying logic of the various manipulations is flimsy. As Jim Hacker said, can you tell me what I don't know :-)?
 
Physics news on Phys.org
  • #2
There are various shortcuts you can use. I'll show some of them via an explicit worked example:
$$[\hat e_\theta \,, \hat e_\phi] ~\equiv~ \Big[ \frac1r \, \partial_\theta ~,~ \frac{1}{r \sin\theta} \, \partial_\phi \Big] ~=~ \frac{1}{r^2} \Big[ \partial_\theta ~,~ \frac{1}{ \sin\theta} \, \partial_\phi \Big] ~.$$In the above, I've moved all the ##r## terms out the front of the commutator because ##r## is independent of ##\theta## and ##\phi##, hence ##r## can pass straight through ##\partial_\theta## and ##\partial_\phi##. (To further check my righthand side above, expand the commutator using the Leibniz product rule ##[A,BC] = B[A,C] + [A,B]C##.)

The next step is $$\frac{1}{r^2} \Big[ \partial_\theta ~,~ \frac{1}{ \sin\theta} \, \partial_\phi \Big] ~=~ \frac{1}{r^2} \Big[ \partial_\theta ~,~ \frac{1}{ \sin\theta} \Big] \, \partial_\phi ~,$$where I've moved ##\partial_\phi## outside of the commutator because ##\theta## and ##\phi## are independent variables. That leaves $$\frac{1}{r^2} \Big[ \partial_\theta ~,~ \frac{1}{ \sin\theta} \Big] \partial_\phi ~=~ \frac{1}{r^2} \;\left[ \partial_\theta \left( \frac{1}{ \sin\theta} \right) \right] \partial_\phi~=~ -\,\frac{\cot\theta}{r^2 \sin\theta}\; \partial_\phi ~=~ -\,\frac{\cot\theta}{r}\; \hat e_\phi ~.$$
If you're having trouble with commutators involving ##\partial## apparently acting on nothing, write the commutator like ##[A,B]f##, where ##f## is an arbitrary function, and expand the commutator explicitly as ##A(B(f)) - B(A(f))##.

HTH.
 
Last edited:
  • Like
Likes vanhees71, chartery, PeroK and 1 other person
  • #3
strangerep said:
I'll show some of them via an explicit worked example
Shouldn't it be ##1 / r## outside the commutator?
 
  • #4
strangerep said:
There are various shortcuts you can use. I'll show some of them via an explicit worked example:
$$[\hat e_\theta \,, \hat e_\phi] ~\equiv~ \Big[ \frac1r \, \partial_\theta ~,~ \frac{1}{r \sin\theta} \, \partial_\phi \Big] ~=~ \frac{1}{r^2} \Big[ \partial_\theta ~,~ \frac{1}{ \sin\theta} \, \partial_\phi \Big] ~.$$In the above, I've moved all the ##r## terms out the front of the commutator because ##r## is independent of ##\theta## and ##\phi##, hence ##r## can pass straight through ##\partial_\theta## and ##\partial_\phi##. (To further check my righthand side above, expand the commutator using the Leibniz product rule ##[A,BC] = B[A,C] + [A,B]C##.)

The next step is $$\frac{1}{r^2} \Big[ \partial_\theta ~,~ \frac{1}{ \sin\theta} \, \partial_\phi \Big] ~=~ \frac{1}{r^2} \Big[ \partial_\theta ~,~ \frac{1}{ \sin\theta} \Big] \, \partial_\phi ~,$$where I've moved ##\partial_\phi## outside of the commutator because ##\theta## and ##\phi## are independent variables. That leaves $$\frac{1}{r^2} \Big[ \partial_\theta ~,~ \frac{1}{ \sin\theta} \Big] \partial_\phi ~=~ \frac{1}{r^2} \; \partial_\theta \left( \frac{1}{ \sin\theta} \right) ~=~ -\,\frac{\cot\theta}{r^2 \sin\theta}\; \partial_\phi ~=~ -\,\frac{\cot\theta}{r}\; \hat e_\phi ~.$$
If you're having trouble with commutators involving ##\partial## apparently acting on nothing, write the commutator like ##[A,B]f##, where ##f## is an arbitrary function, and expand the commutator explicitly as ##A(B(f)) - B(A(f))##.

HTH.
More generally, you can see directly that:
$$[\partial_{\theta}, f(\theta)\partial_{\phi}] = f'(\theta) \partial_{\phi} + f(\theta)\partial_{\theta}\partial_{\phi} - f(\theta)\partial_{\phi}\partial_{\theta} = f'(\theta) \partial_{\phi}$$
 
  • Like
Likes chartery
  • #5
Thanks, that does help fix the chain rule version in my head. But I am uneasy starting on tetrads when the other two (via ##\frac{\partial }{\partial \hat{\theta}}##, or especially the Cartesians) imply I am missing some basic understanding of the manipulations of even the simplest 2D unit polars.
 
  • #6
chartery said:
Thanks, that does help fix the chain rule version in my head. But I am uneasy starting on tetrads when the other two (via ##\frac{\partial }{\partial \hat{\theta}}##, or especially the Cartesians) imply I am missing some basic understanding of the manipulations of even the simplest 2D unit polars.
If you mean this:

chartery said:
Finally, when just multiplying out the commutator of Cartesian versions, everything seems to cancel:
##[\hat{r},\hat{\theta}] = \left( cos\theta\overrightarrow{x}+sin\theta\overrightarrow{y} \right)\left( -sin\theta\overrightarrow{x}+cos\theta\overrightarrow{y} \right)-\left( -sin\theta\overrightarrow{x}+cos\theta\overrightarrow{y} \right)\left( cos\theta\overrightarrow{x}+sin\theta\overrightarrow{y} \right)## =0??Evidently, my grasp of the underlying logic of the various manipulations is flimsy. As Jim Hacker said, can you tell me what I don't know :-)?
That makes little sense. The commutator applies to operators, not to vectors.
 
  • #7
PeroK said:
If you mean this:That makes little sense. The commutator applies to operators, not to vectors.
Yes, I got a bit mixed up between the various e's, thetas, hats, and arrows. It should have been

##[\hat{r},\hat{\theta}] = \left( cos\theta\partial_{x}+sin\theta\partial_{y} \right)\left( -sin\theta\partial_{x}+cos\theta\partial_{y} \right)-\left( -sin\theta\partial_{x}+cos\theta\partial_{y} \right)\left( cos\theta\partial_{x}+sin\theta\partial_{y} \right)##

but the result still seems zero, given commutativity of ##\partial_{x}\partial_{y}## ?
 
  • #8
chartery said:
Yes, I got a bit mixed up between the various e's, thetas, hats, and arrows. It should have been

##[\hat{r},\hat{\theta}] = \left( cos\theta\partial_{x}+sin\theta\partial_{y} \right)\left( -sin\theta\partial_{x}+cos\theta\partial_{y} \right)-\left( -sin\theta\partial_{x}+cos\theta\partial_{y} \right)\left( cos\theta\partial_{x}+sin\theta\partial_{y} \right)##

but the result still seems zero, given commutativity of ##\partial_{x}\partial_{y}## ?
##\theta## is a function of ##x## and ##y##. For example:
$$\cos \theta = \frac{x}{\sqrt{x^2 + y^2}}$$
 
  • Like
Likes chartery
  • #9
Many thanks again. I hope you were not startled by the sound of distant forehead slapping.
 
  • #10
strangerep said:
That leaves $$\frac{1}{r^2} \Big[ \partial_\theta ~,~ \frac{1}{ \sin\theta} \Big] \partial_\phi ~=~ \frac{1}{r^2} \; \partial_\theta \left( \frac{1}{ \sin\theta} \right) ~=~ -\,\frac{\cot\theta}{r^2 \sin\theta}\; \partial_\phi ~=~ -\,\frac{\cot\theta}{r}\; \hat e_\phi ~.$$
HTH.
@strangerep, sorry, I missed you were separate, so forgot to thank you. Now I am working through your reply, I have a question. In the first equality just quoted, what happened to the ##~ \left( \frac{1}{ \sin\theta} \right)\partial_\theta~## term from the commutator? (Assuming the missing ##\partial_\phi## on right is a typo?)
 
  • #11
chartery said:
@strangerep, sorry, I missed you were separate, so forgot to thank you. Now I am working through your reply, I have a question. In the first equality just quoted, what happened to the ##~ \left( \frac{1}{ \sin\theta} \right)\partial_\theta~## term from the commutator? (Assuming the missing ##\partial_\phi## on right is a typo?)
You should work through this yourself. At this level, you need to move away from needing every line in a proof to be fully explained. You need to be able, if necessary, to fill in the blanks yourself.

This seems to be quite a common problem when students move up to advanced undergraduate or graduate level. They are used to being spoon-fed every detail. At a certain level it becomes impossible to give every algebraic step, because there are too many. You need to develop self-sufficiency in this respect.

There's a further clue in my post #2, which you "liked". Hopefully you also understood it!
 
  • #12
chartery said:
@strangerep, sorry, I missed you were separate, so forgot to thank you. Now I am working through your reply, I have a question. In the first equality just quoted, what happened to the ##~ \left( \frac{1}{ \sin\theta} \right)\partial_\theta~## term from the commutator?
##\partial_\theta## acting on nothing can be "shortcut" to zero. This is actually a frequently-used trick. To understand it the first time, make the commutator act on an arbitrary function ##g##, like this: $$[\partial_\theta \,, f(\theta)] g~=~ \partial_\theta fg ~-~ f \partial_\theta g ~=~ (\partial_\theta f) g + f \partial_\theta g ~-~ f \partial_\theta g ~=~ (\partial_\theta f) g ~,$$ where we have used the Leibniz product rule.

Therefore, we can shortcut this to say $$[\partial_\theta \,, f(\theta)] = \partial_\theta f ~.$$
chartery said:
(Assuming the missing ##\partial_\phi## on right is a typo?)
Yes, that was a typo. Now corrected.
 
  • Like
Likes chartery
  • #13
PeterDonis said:
Shouldn't it be ##1 / r## outside the commutator?
I don't think so, unless I'm missing something. Where precisely?
 
  • #14
strangerep said:
##\partial_\theta## acting on nothing can be "shortcut" to zero. This is actually a frequently-used trick. To understand it the first time, make the commutator act on an arbitrary function ##g##, like this: $$[\partial_\theta \,, f(\theta)] g~=~ \partial_\theta fg ~-~ f \partial_\theta g ~=~ (\partial_\theta f) g + f \partial_\theta g ~-~ f \partial_\theta g ~=~ (\partial_\theta f) g ~,$$ where we have used the Leibniz product rule.
Thanks very much for the spoon-feeding. My knowledge is patchy, and being 'woolly' on the flurry of notations, it helps me avoid overlooking subtleties and errors applying generalisations, which of course are obvious in hindsight. (Sorry @PeroK, was deficiency in grounding rather than inclination.). Otherwise I can find later that I didn't understand as well as I thought.
 
  • #15
strangerep said:
I don't think so, unless I'm missing something. Where precisely?
Never mind, I saw what I was missing. You factored ##1 / r^2## out of the commutator because there are two factors of ##1 / r## inside it.
 
  • Like
Likes vanhees71

Similar threads

  • Special and General Relativity
Replies
11
Views
202
  • Special and General Relativity
Replies
8
Views
1K
  • Special and General Relativity
Replies
8
Views
1K
Replies
8
Views
247
  • Special and General Relativity
Replies
4
Views
710
  • Special and General Relativity
Replies
16
Views
2K
  • Special and General Relativity
Replies
5
Views
369
  • Introductory Physics Homework Help
Replies
10
Views
271
  • Introductory Physics Homework Help
Replies
1
Views
905
Replies
47
Views
5K
Back
Top